Difference between revisions of "1962 AHSME Problems/Problem 8"

(Problem)
(Solution)
Line 6: Line 6:
  
 
==Solution==
 
==Solution==
"Unsolved"
+
{{solution}}

Revision as of 22:53, 10 November 2013

Problem

Given the set of $n$ numbers; $n > 1$, of which one is $1 - \frac {1}{n}$ and all the others are $1$. The arithmetic mean of the $n$ numbers is:

$\textbf{(A)}\ 1\qquad\textbf{(B)}\ n-\frac{1}{n}\qquad\textbf{(C)}\ n-\frac{1}{n^2}\qquad\textbf{(D)}\ 1-\frac{1}{n^2}\qquad\textbf{(E)}\ 1-\frac{1}{n}-\frac{1}{n^2}$

Solution

This problem needs a solution. If you have a solution for it, please help us out by adding it.